Set 3 of 3 - Midterm

¡Supera tus tareas y exámenes ahora con Quizwiz!

Which of the following is the primary function of the small intestine?

Absorption

A nurse is caring for a patient who is receiving parenteral nutrition. When writing this patient's plan of care, which of the following nursing diagnoses should be included? A) Risk for Peripheral Neurovascular Dysfunction Related to Catheter Placement B) Ineffective Role Performance Related to Parenteral Nutrition C) Bowel Incontinence Related to Parenteral Nutrition D) Chronic Pain Related to Catheter Placement

Ans: B) Ineffective Role Performance Related to Parenteral Nutrition Feedback: The limitations associated with PN can make it difficult for patients to maintain their usual roles. PN does not normally cause bowel incontinence and catheters are not associated with chronic pain or neurovascular dysfunction.

The nurse is assessing placement of a nasogastric tube that the patient has had in place for 2 days. The tube is draining green aspirate. What is the nurse's most appropriate action? A) Inform the physician that the tube may be in the patient's pleural space. B) Withdraw the tube 2 to 4 cm. C) Leave the tube in its present position. D) Advance the tube up to 8 cm.

Ans: C) Leave the tube in its present position Feedback: The patient's aspirate is from the gastric area when the nurse observes that the color of the aspirate is green. Further confirmation of placement is necessary, but there is likely no need for repositioning. Pleural secretions are pale yellow.

34. A nurse at a long-term care facility is amending the care plan of a resident who has just been diagnosed with essential thrombocythemia (ET). The nurse should anticipate the administration of what medication? A) Dalteparin B) Allopurinol C) Hydroxyurea D) Hydrochlorothiazide

C) Hydroxyurea Hydroxyurea is effective in lowering the platelet count for patients with ET. Dalteparin, allopurinol, and HCTZ do not have this therapeutic effect.

40. A nurse is caring for patient whose diagnosis of multiple myeloma is being treated with bortezomib. The nurse should assess for what adverse effect of this treatment? A) Stomatitis B) Nephropathy C) Cognitive changes D) Peripheral neuropathy

D) Peripheral neuropathy A significant toxicity associated with the use of bortezomib for multiple myeloma is peripheral neuropathy. Stomatitis, cognitive changes, and nephropathy are not noted to be adverse effects of this medication.

After teaching a group of students about the various organs of the upper gastrointestinal tract and possible disorders, the instructor determines that the teaching was successful when the students identify which of the following structures as possibly being affected?

Stomach

While palpating a client's right upper quadrant (RUQ), the nurse would expect to find which structure?

The RUQ contains the liver, gallbladder, duodenum, head of the pancreas, hepatic flexure of the colon, portions of the ascending and transverse colon, and a portion of the right kidney. The sigmoid colon is located in the left lower quadrant; the appendix, in the right lower quadrant; and the spleen, in the left upper quadrant.

The nurse is caring for a client recovering from an esophagogastroduodenoscopy (EGD). Which of the following client symptoms would require further nursing assessment?

The nurse is correct to fully assess the client experiencing abdominal distention following an esophagogastroduodenoscopy (EGD). Abdominal distention could indicate complications such as perforation and bleeding. The client experiences drowsiness from the sedative during the early recovery process and a sore throat from passage of the scope. The client may also experience thirst because the client has not had liquids for a period of time.

The nurse determines a client scheduled to undergo an abdominal ultrasonography should receive which instruction?

A client scheduled to undergo an abdominal ultrasonography should restrict eating of all solid food for 6 to 8 hours to avoid having images of the test obscured with gas and intestinal contents. Ultrasonography records the reflection of sound waves. Strenuous exercises, the consumption of sweets, and exposure to sunlight do not affect the results of the test in any way.

9. A patient with leukemia has developed stomatitis and is experiencing a nutritional deficit. An oral anesthetic has consequently been prescribed. What health education should the nurse provide to the patient? A) Chew with care to avoid inadvertently biting the tongue. B) Use the oral anesthetic 1 hour prior to meal time. C) Brush teeth before and after eating. D) Swallow slowly and deliberately.

A) Chew with care to avoid inadvertently biting the tongue. If oral anesthetics are used, the patient must be warned to chew with extreme care to avoid inadvertently biting the tongue or buccal mucosa. An oral anesthetic would be metabolized by the time the patient eats if it is used 1 hour prior to meals. There is no specific need to warn the patient about brushing teeth or swallowing slowly because an oral anesthetic has been used.

35. A nurse is writing the care plan of a patient who has been diagnosed with myelofibrosis. What nursing diagnoses should the nurse address? Select all that apply. A) Disturbed Body Image B) Impaired Mobility C) Imbalanced Nutrition: Less than Body Requirements D) Acute Confusion E) Risk for Infection

A) Disturbed Body Image B) Impaired Mobility C) Imbalanced Nutrition: Less than Body Requirements E) Risk for Infection The profound splenomegaly that accompanies myelofibrosis can impact the patient's body image and mobility. As well, nutritional deficits are common and the patient is at risk for infection. Cognitive effects are less common.

25. An oncology nurse recognizes a patient's risk for fluid imbalance while the patient is undergoing treatment for leukemia. What relevant assessments should the nurse include in the patient's plan of care? Select all that apply. A) Monitoring the patient's electrolyte levels B) Monitoring the patient's hepatic function C) Measuring the patient's weight on a daily basis D) Measuring and recording the patient's intake and output E) Auscultating the patient's lungs frequently

A) Monitoring the patient's electrolyte levels C) Measuring the patient's weight on a daily basis D) Measuring and recording the patient's intake and output E) Auscultating the patient's lungs frequently Assessments that relate to fluid balance include monitoring the patient's electrolytes, auscultating the patient's chest for adventitious sounds, weighing the patient daily, and closely monitoring intake and output. Liver function is not directly relevant to the patient's fluid status in most cases.

22. A patient has been found to have an indolent neoplasm. The nurse should recognize what implication of this condition? A) The patient faces a significant risk of malignancy. B) The patient has a myeloid form of leukemia. C) The patient has a lymphocytic form of leukemia. D) The patient has a major risk factor for hemophilia.

A) The patient faces a significant risk of malignancy. Indolent neoplasms have the potential to develop into a neoplasm, but this is not always the case. The patient does not necessary have, or go on to develop, leukemia. Indolent neoplasms are unrelated to the pathophysiology of hemophilia.

28. Following an extensive diagnostic workup, an older adult patient has been diagnosed with a secondary myelodysplastic syndrome (MDS). What assessment question most directly addresses the potential etiology of this patient's health problem? A) Were you ever exposed to toxic chemicals in any of the jobs that you held? B) When you were younger, did you tend to have recurrent infections of any kind? C) Have your parents or siblings had any disease like this? D) Would you say that you've had a lot of sun exposure in your lifetime?"

A) Were you ever exposed to toxic chemicals in any of the jobs that you held? Secondary MDS can occur at any age and results from prior toxic exposure to chemicals, including chemotherapeutic medications. Family history, sun exposure, and previous infections are unrelated to the pathophysiology of secondary MDS.

A nurse who provides care in an ambulatory clinic integrates basic cancer screening into admission assessments. What patient most likely faces the highest immediate risk of oral cancer? A) A 65-year-old man with alcoholism who smokes B) A 45-year-old woman who has type 1 diabetes and who wears dentures C) A 32-year-old man who is obese and uses smokeless tobacco D) A 57-year-old man with GERD and dental caries

Ans: A) A 65- year-old man with alcoholism who smokes Feedback: Oral cancers are often associated with the use of alcohol and tobacco, which when used together have a synergistic carcinogenic effect. Most cases of oral cancers occur in people over the age of 60 and a disproportionate number of cases occur in men. Diabetes, dentures, dental caries, and GERD are not risk factors for oral cancer.

A nurse is caring for a patient who has an order to discontinue the administration of parenteral nutrition. What should the nurse do to prevent the occurrence of rebound hypoglycemia in the patient? A) Administer an isotonic dextrose solution for 1 to 2 hours after discontinuing the PN. B) Administer a hypertonic dextrose solution for 1 to 2 hours after discontinuing the PN. C) Administer 3 ampules of dextrose 50% immediately prior to discontinuing the PN. D) Administer 3 ampules of dextrose 50% 1 hour after discontinuing the PN.

Ans: A) Administer an isotonic dextrose solution for 1 to 2 hours after discontinuing the PN Feedback: After administration of the PN solution is gradually discontinued, an isotonic dextrose solution is administered for 1 to 2 hours to protect against rebound hypoglycemia. The other listed actions would likely cause hyperglycemia.

A patient has been discharged home on parenteral nutrition (PN). Much of the nurse's discharge education focused on coping. What must a patient on PN likely learn to cope with? Select all that apply. A) Changes in lifestyle B) Loss of eating as a social behavior C) Chronic bowel incontinence from GI changes D) Sleep disturbances related to frequent urination during nighttime infusions E) Stress of choosing the correct PN formulation

Ans: A) Changes in lifestyle B) Loss of eating as a social behavior D) Sleep disturbances related to frequent urination during nighttime infusions Feedback: Patients must cope with the loss of eating as a social behavior and with changes in lifestyle brought on by sleep disturbances related to frequent urination during night time infusions. PN is not associated with bowel incontinence and the patient does not select or adjust the formulation of PN.

A patient's health decline necessitates the use of total parenteral nutrition. The patient has questioned the need for insertion of a central venous catheter, expressing a preference for a normal IV. The nurse should know that peripheral administration of high-concentration PN formulas is contraindicated because of the risk for what complication? A) Chemical phlebitis B) Hyperglycemia C) Dumping syndrome D) Line sepsis

Ans: A) Chemical phlebitis Feedback: Formulations with dextrose concentrations of more than 10% should not be administered through peripheral veins because they irritate the intima (innermost walls) of small veins, causing chemical phlebitis. Hyperglycemia and line sepsis are risks with both peripheral and central administration of PN. PN is not associated with dumping syndrome.

A nurse is preparing to administer a patient's intravenous fat emulsion simultaneously with parenteral nutrition (PN). Which of the following principles should guide the nurse's action? A) Intravenous fat emulsions may be infused simultaneously with PN through a Y-connector close to the infusion site and should not be filtered. B) The nurse should prepare for placement of another intravenous line, as intravenous fat emulsions may not be infused simultaneously through the line used for PN. C) Intravenous fat emulsions may be infused simultaneously with PN through a Y-connector close to the infusion site after running the emulsion through a filter. D) The intravenous fat emulsions can be piggy-backed into any existing IV solution that is infusing.

Ans: A) Intravenous fat emulsions may be infused simultaneously with PN through a Y-connector close to the infusion site and should not be filtered Feedback: Intravenous fat emulsions may be infused simultaneously with PN through a Y-connector close to the infusion site and should not be filtered. The patient does not need another intravenous line for the fat emulsion. The IVFE cannot be piggy-backed into any existing IV solution that is infusing.

A patient has been diagnosed with achalasia based on his history and diagnostic imaging results. The nurse should identify what risk diagnosis when planning the patients care? A) Risk for Aspiration Related to Inhalation of Gastric Contents B) Risk for Imbalanced Nutrition: Less than Body Requirements Related to Impaired Absorption C) Risk for Decreased Cardiac Output Related to Vasovagal Response D) Risk for Impaired Verbal Communication Related to Oral Trauma

Ans: A) Risk for Aspiration Related to Inhalation of Gastric Contents Feedback: Achalasia can result in the aspiration of gastric contents. It is not normally an acute risk to the patients nutritional status and does not affect cardiac output or communication.

A patients neck dissection surgery resulted in damage to the patients superior laryngeal nerve. What area of assessment should the nurse consequently prioritize? A) The patients swallowing ability B) The patients ability to speak C) The patients management of secretions D) The patients airway patency

Ans: A) The patients swallowing ability Feedback: If the superior laryngeal nerve is damaged, the patient may have difficulty swallowing liquids and food because of the partial lack of sensation of the glottis. Damage to this particular nerve does not inhibit speech and only affects management of secretions and airway patency indirectly.

A staff educator is reviewing the causes of gastroesophageal reflux disease (GERD) with new staff nurses. What area of the GI tract should the educator identify as the cause of reduced pressure associated with GERD? A) Pyloric sphincter B) Lower esophageal sphincter C) Hypopharyngeal sphincter D) Upper esophageal sphincter

Ans: B) Lower esophageal sphincter Feedback: The lower esophageal sphincter, also called the gastroesophageal sphincter or cardiac sphincter, is located at the junction of the esophagus and the stomach. An incompetent lower esophageal sphincter allows reflux (backward flow) of gastric contents. The upper esophageal sphincter and the hypopharyngeal sphincter are synonymous and are not responsible for the manifestations of GERD. The pyloric sphincter exists between the stomach and the duodenum.

A nursing educator is reviewing the care of patients with feeding tubes and endotracheal tubes (ET). The educator has emphasized the need to check for tube placement in the stomach as well as residual volume. What is the main purpose of this nursing action? A) Prevent gastric ulcers B) Prevent aspiration C) Prevent abdominal distention D) Prevent diarrhea

Ans: B) Prevent aspiration Feedback: Protecting the client from aspirating is essential because aspiration can cause pneumonia, a potentially life-threatening disorder. Gastric ulcers are not a common complication of tube feeding in clients with ET tubes. Abdominal distention and diarrhea can both be associated with tube feeding, but prevention of these problems is not the primary rationale for confirming placement.

You are caring for a patient who was admitted to have a low-profile gastrostomy device (LPGD) placed. How soon after the original gastrostomy tube placement can an LPGD be placed? A) 2 weeks B) 4 to 6 weeks C) 2 to 3 months D) 4 to 6 months

Ans: C) 2 to 3 months Feedback: An alternative to the PEG device is a low-profile gastrostomy device (LPGD). LPGDs may be inserted 2 to 3 months after initial gastrostomy tube placement.

A nurse is providing care for a patient whose neck dissection surgery involved the use of a graft. When assessing the graft, the nurse should prioritize data related to what nursing diagnosis? A) Risk for Disuse Syndrome B) Unilateral Neglect C) Risk for Trauma D) Ineffective Tissue Perfusion

Ans: D) Ineffective Tissue Perfusion Feedback: Grafted skin is highly vulnerable to inadequate perfusion and subsequent ischemia and necrosis. This is a priority over chronic pain, which is unlikely to be a long-term challenge. Neglect and disuse are not risks related to the graft site.

A nurse is caring for a patient who is postoperative day 1 following neck dissection surgery. The nurse is performing an assessment of the patient and notes the presence of high-pitched adventitious sounds over the patients trachea on auscultation. The patients oxygen saturation is 90% by pulse oximetry with a respiratory rate of 31 breaths per minute. What is the nurses most appropriate action? A) Encourage the patient to perform deep breathing and coughing exercises hourly. B) Reposition the patient into a prone or semi-Fowlers position and apply supplementary oxygen by nasal cannula. C) Activate the emergency response system. D) Report this finding promptly to the physician and remain with the patient.

Ans: D) Report this finding promptly to the physician and remain with the patient Feedback: In the immediate postoperative period, the nurse assesses for stridor (coarse, high-pitched sound on inspiration) by listening frequently over the trachea with a stethoscope. This finding must be reported immediately because it indicates obstruction of the airway. The patients current status does not warrant activation of the emergency response system, and encouraging deep breathing and repositioning the patient are inadequate responses.

A nurse has obtained an order to remove a patient's NG tube and has prepared the patient accordingly. After flushing the tube and removing the nasal tape, the nurse attempts removal but is met with resistance. Because the nurse is unable to overcome this resistance, what is the most appropriate action? A) Gently twist the tube before pulling. B) Instill a digestive enzyme solution and reattempt removal in 10 to 15 minutes. C) Flush the tube with hot tap water and reattempt removal. D) Report this finding to the patient's primary care provider.

Ans: D) Report this finding to the patient's primary care provider Feedback: If the tube does not come out easily, force should not be used, and the problem should be reported to the primary provider. Enzymes are used to resolve obstructions, not to aid removal. For safety reasons, hot water is never instilled into a tube. Twisting could cause damage to the mucosa.

A nurse is creating a care plan for a patient with a nasogastric tube. How should the nurse direct other members of the care team to check correct placement of the tube? A) Auscultate the patient's abdomen after injecting air through the tube. B) Assess the color and pH of aspirate. C) Locate the marking made after the initial x-ray confirming placement. D) Use a combination of at least two accepted methods for confirming placement.

Ans: D) Use a combination of at least two accepted methods for confirming placement Feedback: There are a variety of methods to check tube placement. The safest way to confirm placement is to utilize a combination of assessment methods.

39. A patient has a diagnosis of multiple myeloma and the nurse is preparing health education in preparation for discharge from the hospital. What action should the nurse promote? A) Daily performance of weight-bearing exercise to prevent muscle atrophy B) Close monitoring of urine output and kidney function C) Daily administration of warfarin (Coumadin) as ordered D) Safe use of supplementary oxygen in the home setting

B) Close monitoring of urine output and kidney function Renal function must be monitored closely in the patient with multiple myeloma. Excessive weight-bearing can cause pathologic fractures. There is no direct indication for anticoagulation or supplementary oxygen.

30. A nurse is preparing health education for a patient who has received a diagnosis of myelodysplastic syndrome (MDS). Which of the following topics should the nurse prioritize? A) Techniques for energy conservation and activity management B) Emergency management of bleeding episodes C) Technique for the administration of bronchodilators by metered-dose inhaler D) Techniques for self-palpation of the lymph nodes

B) Emergency management of bleeding episodes Because of patients' risks of hemorrhage, patients with MDS should be taught techniques for managing emergent bleeding episodes. Bronchodilators are not indicated for the treatment of MDS and lymphedema is not normally associated with the disease. Energy conservation techniques are likely to be useful, but management of hemorrhage is a priority because of the potential consequences.

10. A patient diagnosed with acute myelogenous leukemia has just been admitted to the oncology unit. When writing this patient's care plan, what potential complication should the nurse address? A) Pancreatitis B) Hemorrhage C) Arteritis D) Liver dysfunction

B) Hemorrhage Pancreatitis, arteritis, and liver dysfunction are generally not complications of leukemia. However, the patient faces a high risk of hemorrhage.

21. An older adult patient is undergoing diagnostic testing for chronic lymphocytic leukemia (CLL). What assessment finding is certain to be present if the patient has CLL? A) Increased numbers of blast cells B) Increased lymphocyte levels C) Intractable bone pain D) Thrombocytopenia with no evidence of bleeding

B) Increased lymphocyte levels An increased lymphocyte count (lymphocytosis) is always present in patients with CLL. Each of the other listed symptoms may or may not be present, and none is definitive for CLL.

38. The clinical nurse educator is presenting health promotion education to a patient who will be treated for non-Hodgkin lymphoma on an outpatient basis. The nurse should recommend which of the following actions? A) Avoiding direct sun exposure in excess of 15 minutes daily B) Avoiding grapefruit juice and fresh grapefruit C) Avoiding highly crowded public places D) Using an electric shaver rather than a razor

C) Avoiding highly crowded public places The risk of infection is significant for these patients, not only from treatment-related myelosuppression but also from the defective immune response that results from the disease itself. Limiting infection exposure is thus necessary. The need to avoid grapefruit is dependent on the patient's medication regimen. Sun exposure and the use of razors are not necessarily contraindicated.

37. A young adult patient has received the news that her treatment for Hodgkin lymphoma has been deemed successful and that no further treatment is necessary at this time. The care team should ensure that the patient receives regular health assessments in the future due to the risk of what complication? A) Iron-deficiency anemia B) Hemophilia C) Hematologic cancers D) Genitourinary cancers

C) Hematologic cancers Survivors of Hodgkin lymphoma have a high risk of second cancers, with hematologic cancers being the most common. There is no consequent risk of anemia or hemophilia, and hematologic cancers are much more common than GU cancers.

11. An emergency department nurse is triaging a 77-year-old man who presents with uncharacteristic fatigue as well as back and rib pain. The patient denies any recent injuries. The nurse should recognize the need for this patient to be assessed for what health problem? A) Hodgkin disease B) Non-Hodgkin lymphoma C) Multiple myeloma D) Acute thrombocythemia

C) Multiple myeloma Back pain, which is often a presenting symptom in multiple myeloma, should be closely investigated in older patients. The lymphomas and bleeding disorders do not typically present with the primary symptom of back pain or rib pain.

29. A patient with a myelodysplastic syndrome is being treated on the medical unit. What assessment finding should prompt the nurse to contact the patient's primary care provider? A) The patient is experiencing a frontal lobe headache. B) The patient has an episode of urinary incontinence. C) The patient has an oral temperature of 37.5ºC (99.5ºF). D) The patient's SpO2 is 91% on room air.

C) The patient has an oral temperature of 37.5ºC (99.5ºF). Because the patient with MDS is at a high risk for infection, any early signs of infection must be reported promptly. The nurse should address each of the listed assessment findings, but none is as direct a threat to the patient's immediate health as an infection.

8. A 35-year-old male is admitted to the hospital complaining of severe headaches, vomiting, and testicular pain. His blood work shows reduced numbers of platelets, leukocytes, and erythrocytes, with a high proportion of immature cells. The nurse caring for this patient suspects a diagnosis of what? A) AML B) CML C) MDS D) ALL

D) ALL In acute lymphocytic leukemia (ALL), manifestations of leukemic cell infiltration into other organs are more common than with other forms of leukemia, and include pain from an enlarged liver or spleen, as well as bone pain. The central nervous system is frequently a site for leukemic cells; thus, patients may exhibit headache and vomiting because of meningeal involvement. Other extranodal sites include the testes and breasts. This particular presentation is not closely associated with acute myeloid leukemia (AML), chronic myeloid leukemia (CML), or myelodysplastic syndromes (MDS)

17. Diagnostic testing has resulted in a diagnosis of acute myeloid leukemia (AML) in an adult patient who is otherwise healthy. The patient and the care team have collaborated and the patient will soon begin induction therapy. The nurse should prepare the patient for which of the following? A) Daily treatment with targeted therapy medications B) Radiation therapy on a daily basis C) Hematopoietic stem cell transplantation D) An aggressive course of chemotherapy

D) An aggressive course of chemotherapy Attempts are made to achieve remission of AML by the aggressive administration of chemotherapy, called induction therapy, which usually requires hospitalization for several weeks. Induction therapy is not synonymous with radiation, stem cell transplantation, or targeted therapies.

31. A clinic patient is being treated for polycythemia vera and the nurse is providing health education. What practice should the nurse recommend in order to prevent the complications of this health problem? A) Avoiding natural sources of vitamin K B) Avoiding altitudes of ³1500 feet (457 meters) C) Performing active range of motion exercises daily D) Avoiding tight and restrictive clothing on the legs

D) Avoiding tight and restrictive clothing on the legs Because of the risk of DVT, patients with polycythemia vera should avoid tight and restrictive clothing. There is no need to avoid foods with vitamin K or to avoid higher altitudes. Activity levels should be maintained, but there is no specific need for ROM exercises.

12. A home health nurse is caring for a patient with multiple myeloma. Which of the following interventions should the nurse prioritize when addressing the patient's severe bone pain? A) Implementing distraction techniques B) Educating the patient about the effective use of hot and cold packs C) Teaching the patient to use NSAIDs effectively D) Helping the patient manage the opioid analgesic regime

D) Helping the patient manage the opioid analgesic regime For severe pain resulting from multiple myeloma, opioids are likely necessary. NSAIDs would likely be ineffective and are associated with significant adverse effects. Hot and cold packs as well as distraction would be insufficient for severe pain.

27. A patient has completed the full course of treatment for acute lymphocytic leukemia and has failed to respond appreciably. When preparing for the patient's subsequent care, the nurse should perform what action? A) Arrange a meeting between the patient's family and the hospital chaplain. B) Assess the factors underlying the patient's failure to adhere to the treatment regimen. C) Encourage the patient to vigorously pursue complementary and alternative medicine (CAM). D) Identify the patient's specific wishes around end-of-life care.

D) Identify the patient's specific wishes around end-of-life care Should the patient not respond to therapy, it is important to identify and respect the patient's choices about treatment, including measures to prolong life and other end-of-life measures. The patient may or may not be open to pursuing CAM. Unsuccessful treatment is not necessarily the result of failure to adhere to the treatment plan. Assessment should precede meetings with a chaplain, which may or may not be beneficial to the patient and congruent with the family's belief system.

14. A patient with non-Hodgkin's lymphoma is receiving information from the oncology nurse. The patient asks the nurse why she should stop drinking and smoking and stay out of the sun. What would be the nurse's best response? A) Everyone should do these things because they're health promotion activities that apply to everyone. B) You don't want to develop a second cancer, do you? C) You need to do this just to be on the safe side. D) It's important to reduce other factors that increase the risk of second cancers.

D) It's important to reduce other factors that increase the risk of second cancers. The nurse should encourage patients to reduce other factors that increase the risk of developing second cancers, such as use of tobacco and alcohol and exposure to environmental carcinogens and excessive sunlight. The other options do not answer the patient's question, and also make light of the patient's question.

The nurse determines one or two bowel sounds in 2 minutes should be documented as

Hypoactive bowel sound is the description given to auscultation of one to two bowel sounds in 2 minutes. Normal bowel sounds are heard every 5 to 20 seconds. Hyperactive bowel sounds occur when 5 or 6 sounds are heard in less than 30 seconds. The nurse records that bowel sounds are absent when no sound is heard in 3 to 5 minutes.

Specific disease processes and ingestion of certain foods and medications may change the appearance of the stool. If blood is shed in sufficient quantities into the upper gastrointestinal (GI) tract, it produces which change in the stool appearance?

If the blood is shed in sufficient quantities into the upper GI tract, it produces a tarry-black color, whereas blood entering the lower portion of the GI tract or passing rapidly though will cause the stool to appear bright or dark red. Lower rectal or anal bleeding is suspected if there is streaking of the blood on the surface of the stool or if blood is noted on toilet tissue. Stool is normally light or dark brown.

Which term describes a gastric secretion that combines with vitamin B12 so that it can be absorbed?

Intrinsic factor, secreted by the gastric mucosa, combines with dietary vitamin B12 so that the vitamin can be absorbed in the ileum. In the absence of intrinsic factor, vitamin B12 cannot be absorbed and pernicious anemia results. Amylase is an enzyme that aids in the digestion of starch. Pepsin, an important enzyme for protein digestion, is the end product of the conversion of pepsinogen from the chief cells. Digestive enzymes secreted by the pancreas include trypsin, which aids in digesting protein.

When bowel sounds are heard about every 15 seconds, the nurse would record that the bowel sounds are

Normal bowel sounds are heard every 5 to 20 seconds. Hypoactive bowel sound is the description given to auscultation of one to two bowel sounds in 2 minutes. Sluggish is not a term a nurse would use to accurately describe bowel sounds. The nurse records that bowel sounds are absent when no sound is heard in 3 to 5 minutes.

The nurse asks a client to point to where she feels pain. The client asks why this is important. The nurse's best response would be which of the following?

Pain can be a major symptom of disease. The location and distribution of pain can be referred from a different area. If a client points to an area of pain and has other symptoms associated with a certain disease, this is valuable information for treatment.

The nurse is working with clients with digestive tract disorders. Which of the following organs does the nurse realize has effects as an exocrine gland and an endocrine gland?

The pancreas is both an exocrine gland, one that releases secretions into a duct or channel, and an endocrine gland, one that releases substances directly into the bloodstream. The other organs have a variety of functions but do not have a combination function such as the pancreas.

The nurse determines which is a true statement regarding older clients, considering the age-related effects on their GI system?

They tend usually to have less control of the rectal sphincter.

13. A nurse is caring for a patient with Hodgkin lymphoma at the oncology clinic. The nurse should be aware of what main goal of care? A) Cure of the disease B) Enhancing quality of life C) Controlling symptoms D) Palliation

A) Cure of the disease The goal in the treatment of Hodgkin lymphoma is cure. Palliation is thus not normally necessary. Quality of life and symptom control are vital, but the overarching goal is the cure the disease.

3. An oncology nurse is caring for a patient with multiple myeloma who is experiencing bone destruction. When reviewing the patient's most recent blood tests, the nurse should anticipate what imbalance? A) Hypercalcemia B) Hyperproteinemia C) Elevated serum viscosity D) Elevated RBC count

A) Hypercalcemia Hypercalcemia may result when bone destruction occurs due to the disease process. Elevated serum viscosity occurs because plasma cells excrete excess immunoglobulin. RBC count will be decreased. Hyperproteinemia would not be present.

2. A nurse is caring for a patient who has a diagnosis of acute leukemia. What assessment most directly addresses the most common cause of death among patients with leukemia? A) Monitoring for infection B) Monitoring nutritional status C) Monitor electrolyte levels D) Monitoring liver function

A) Monitoring for infection In patients with acute leukemia, death typically occurs from infection or bleeding. Compromised nutrition, electrolyte imbalances, and impaired liver function are all plausible, but none is among the most common causes of death in this patient population.

18. A patient with a diagnosis of acute myeloid leukemia (AML) is being treated with induction therapy on the oncology unit. What nursing action should be prioritized in the patient's care plan? A) Protective isolation and vigilant use of standard precautions B) Provision of a high-calorie, low-texture diet and appropriate oral hygiene C) Including the family in planning the patient's activities of daily living D) Monitoring and treating the patient's pain

A) Protective isolation and vigilant use of standard precautions Induction therapy causes neutropenia and a severe risk of infection. This risk must be addressed directly in order to ensure the patient's survival. For this reason, infection control would be prioritized over nutritional interventions, family care, and pain, even though each of these are important aspects of nursing care.

33. A nurse is planning the care of a patient who has been diagnosed with essential thrombocythemia (ET). What nursing diagnosis should the nurse prioritize when choosing interventions? A) Risk for Ineffective Tissue Perfusion B) Risk for Imbalanced Fluid Volume C) Risk for Ineffective Breathing Pattern D) Risk for Ineffective Thermoregulation

A) Risk for Ineffective Tissue Perfusion Patients with ET are at risk for hypercoagulation and consequent ineffective tissue perfusion. Fluid volume, breathing, and thermoregulation are not normally affected

1.An oncology nurse is providing health education for a patient who has recently been diagnosed with leukemia. What should the nurse explain about commonalities between all of the different subtypes of leukemia? A) The different leukemias all involve unregulated proliferation of white blood cells. B) The different leukemias all have unregulated proliferation of red blood cells and decreased bone marrow function. C) The different leukemias all result in a decrease in the production of white blood cells. D) The different leukemias all involve the development of cancer in the lymphatic system.

A) The different leukemias all involve unregulated proliferation of white blood cells. Leukemia commonly involves unregulated proliferation of white blood cells. Decreased production of red blood cells is associated with anemias. Decreased production of white blood cells is associated with leukopenia. The leukemias are not characterized by their involvement with the lymphatic system.

20. A 60-year-old patient with chronic myeloid leukemia will be treated in the home setting and the nurse is preparing appropriate health education. What topic should the nurse emphasize? A) The importance of adhering to the prescribed drug regimen B) The need to ensure that vaccinations are up to date C) The importance of daily physical activity D) The need to avoid shellfish and raw foods

A) The importance of adhering to the prescribed drug regimen Nurses need to understand that the effectiveness of the drugs used to treat CML is based on the ability of the patient to adhere to the medication regimen as prescribed. Adherence is often incomplete, thus this must be a focus of health education. Vaccinations normally would not be administered during treatment and daily physical activity may be impossible for the patient. Dietary restrictions are not normally necessary.

The nurse notes that a patient who has undergone skin, tissue, and muscle grafting following a modified radical neck dissection requires suctioning. What is the most important consideration for the nurse when suctioning this patient? A) Avoid applying suction on or near the suture line. B) Position patient on the non operative side with the head of the bed down. C) Assess the patients ability to perform self-suctioning. D) Evaluate the patients ability to swallow saliva and clear fluids.

Ans: A) Avoid applying suction on or near the suture line Feedback: The nurse should avoid positioning the suction catheter on or near the graft suture lines. Application of suction in these areas could damage the graft. Self-sectioning may be unsafe because the patient may damage the suture line. Following a modified radical neck dissection with graft, the patient is usually positioned with the head of the bed elevated to promote drainage and reduce edema. Assessing viability of the graft is important but is not part of the suctioning procedure and may delay initiating suctioning. Maintenance of a patent airway is a nursing priority. Similarly, the patients ability to swallow is an important assessment for the nurse to make; however, it is not directly linked to the patients need for suctioning.

The nurse is administering total parenteral nutrition (TPN) to a client who underwent surgery for gastric cancer. Which of the nurse's assessments most directly addresses a major complication of TPN? A) Checking the patient's capillary blood glucose levels regularly B) Having the patient frequently rate his or her hunger on a 10-point scale C) Measuring the patient's heart rhythm at least every 6 hours D) Monitoring the patient's level of consciousness each shift

Ans: A) Checking the patient's capillary blood glucose levels regularly Feedback: The solution, used as a base for most TPN, consists of a high dextrose concentration and may raise blood glucose levels significantly, resulting in hyperglycemia. This is a more salient threat than hunger, though this should be addressed. Dysrhythmias and decreased LOC are not among the most common complications.

A patient's new onset of dysphagia has required insertion of an NG tube for feeding; the nurse has modified the patient's care plan accordingly. What intervention should the nurse include in the patient's plan of care? A) Confirm placement of the tube prior to each medication administration. B) Have the patient sip cool water to stimulate saliva production. C) Keep the patient in a low Fowler's position when at rest. D) Connect the tube to continuous wall suction when not in use.

Ans: A) Confirm placement of the tube prior to each medication administration Feedback: Each time liquids or medications are administered, and once a shift for continuous feedings, the tube must be checked to ensure that it remains properly placed. If the NG tube is used for decompression, it is attached to intermittent low suction. During the placement of a nasogastric tube the patient should be positioned in a Fowler's position. Oral fluid administration is contraindicated by the patient's dysphagia.

A nurse is caring for a patient who is postoperative from a neck dissection. What would be the most appropriate nursing action to enhance the patients appetite? A) Encourage the family to bring in the patients favored foods. B) Limit visitors at mealtimes so that the patient is not distracted. C) Avoid offering food unless the patient initiates. D) Provide thorough oral care immediately after the patient eats.

Ans: A) Encourage the family to bring in the patients favored foods Feedback: Family involvement and home-cooked favorite foods may help the patient to eat. Having visitors at mealtimes may make eating more pleasant and increase the patients appetite. The nurse should not place the complete onus for initiating meals on the patient. Oral care after meals is necessary, but does not influence appetite.

A nurse is caring for a patient who has had surgery for oral cancer. When addressing the patients long-term needs, the nurse should prioritize interventions and referrals with what goal? A) Enhancement of verbal communication B) Enhancement of immune function C) Maintenance of adequate social support D) Maintenance of fluid balance

Ans: A) Enhancement of verbal communication Feedback: Verbal communication may be impaired by radical surgery for oral cancer. Addressing this impairment often requires a long-term commitment. Immune function, social support, and fluid balance are all necessary, but communication is a priority issue for patients recovering from this type of surgery.

A patient has received treatment for oral cancer. The combination of medications and radiotherapy has resulted in leukopenia. Which of the following is an appropriate response to this change in health status? A) Ensure that none of the patients visitors has an infection. B) Arrange for a diet that is high in protein and low in fat. C) Administer colony stimulating factors (CSFs) as ordered. D) Prepare to administer chemotherapeutics as ordered.

Ans: A) Ensure that none of the patients visitors has an infection Feedback: Leukopenia reduces defense mechanisms, increasing the risk of infections. Visitors who might transmit microorganisms are prohibited if the patients immunologic system is depressed. Changes in diet, CSFs, and the use of chemotherapy do not resolve leukopenia.

A medical nurse who is caring for a patient being discharged home after a radical neck dissection has collaborated with the home health nurse to develop a plan of care for this patient. What is a priority psychosocial outcome for a patient who has had a radical neck dissection? A) Indicates acceptance of altered appearance and demonstrates positive self-image B) Freely expresses needs and concerns related to postoperative pain management C) Compensates effectively for alteration in ability to communicate related to dysarthria D) Demonstrates effective stress management techniques to promote muscle relaxation

Ans: A) Indicates acceptance of altered appearance and demonstrates positive self-image Feedback: Since radical neck dissection involves removal of the sternocleidomastoid muscle, spinal accessory muscles, and cervical lymph nodes on one side of the neck, the patients appearance is visibly altered. The face generally appears asymmetric, with a visible neck depression; shoulder drop also occurs frequently. These changes have the potential to negatively affect self-concept and body image. Facilitating adaptation to these changes is a crucial component of nursing intervention. Patients who have had head and neck surgery generally report less pain as compared with other postoperative patients; however, the nurse must assess each individual patients level of pain and response to analgesics. Patients may experience transient hoarseness following a radical neck dissection; however, their ability to communicate is not permanently altered. Stress management is beneficial but would not be considered the priority in this clinical situation.

A patient has a gastrostomy tube that has been placed to drain stomach contents by low intermittent suction. What is the nurse's priority during this aspect of the patient's care? A) Measure and record drainage. B) Monitor drainage for change in color. C) Titrate the suction every hour. D) Feed the patient via the G tube as ordered.

Ans: A) Measure and record drainage Feedback: This drainage should be measured and recorded because it is a significant indicator of GI function. The nurse should indeed monitor the color of the output, but fluid balance is normally the priority. Frequent titration of the suction should not be necessary and feeding is contraindicated if the G tube is in place for drainage.

A patient with GERD has undergone diagnostic testing and it has been determined that increasing the pace of gastric emptying may help alleviate symptoms. The nurse should anticipate that the patient may be prescribed what drug? A) Metoclopramide (Reglan) B) Omeprazole (Prilosec) C) Lansoprazole (Prevacid) D) Famotidine (Pepcid)

Ans: A) Metoclopramide (Reglan) Feedback: Metoclopramide (Reglan) is useful in promoting gastric motility. Omeprazole and lansoprozole are proton pump inhibitors that reduce gastric acid secretion. Famotidine (Pepcid) is an H2receptor antagonist, which has a similar effect.

The school nurse is planning a health fair for a group of fifth graders and dental health is one topic that the nurse plans to address. What would be most likely to increase the risk of tooth decay? A) Organic fruit juice B) Roasted nuts C) Red meat that is high in fat D) Cheddar cheese

Ans: A) Organic fruit juice Feedback: Dental caries may be prevented by decreasing the amount of sugar and starch in the diet. Patients who snack should be encouraged to choose less cariogenic alternatives, such as fruits, vegetables, nuts, cheeses, or plain yogurt. Fruit juice is high in sugar, regardless of whether it is organic.

An elderly patient comes into the emergency department complaining of an earache. The patient and has an oral temperature of 100.2F and otoscopic assessment of the ear reveals a pearly gray tympanic membrane with no evidence of discharge or inflammation. Which action should the triage nurse take next? A) Palpate the patients parotid glands to detect swelling and tenderness. B) Assess the temporomandibular joint for evidence of a malocclusion. C) Test the integrity of cranial nerve XII by asking the patient to protrude the tongue. D) Inspect the patients gums for bleeding and hyperpigmentation.

Ans: A) Palpate the patients parotid glands to detect swelling and tenderness Feedback: Older adults and debilitated patients of any age who are dehydrated or taking medications that reduce saliva production are at risk for parotitis. Symptoms include fever and tenderness, as well as swelling of the parotid glands. Pain radiates to the ear. Pain associated with malocclusion of the temporomandibular joint may also radiate to the ears; however, a temperature elevation would not be associated with malocclusion. The 12th cranial nerve is not associated with the auditory system. Bleeding and hyperpigmented gums may be caused by pyorrhea or gingivitis. These conditions do not cause earache; fever would not be present unless the teeth were abscessed.

A nurse is caring for a patient in the late stages of esophageal cancer. The nurse should plan to prevent or address what characteristics of this stage of the disease? Select all that apply. A) Perforation into the mediastinum B) Development of an esophageal lesion C) Erosion into the great vessels D) Painful swallowing E) Obstruction of the esophagus

Ans: A) Perforation into the mediastinum C) Erosion into the great vessels E) Obstruction of the esophagus Feedback: In the later stages of esophageal cancer, obstruction of the esophagus is noted, with possible perforation into the mediastinum and erosion into the great vessels. Painful swallowing and the emergence of a lesion are early signs of esophageal cancer.

A nurse is preparing to place a patient's ordered nasogastric tube. How should the nurse best determine the correct length of the nasogastric tube? A) Place distal tip to nose, then ear tip and end of xiphoid process. B) Instruct the patient to lie prone and measure tip of nose to umbilical area. C) Insert the tube into the patient's nose until secretions can be aspirated. D) Obtain an order from the physician for the length of tube to insert.

Ans: A) Place distal top to nose, then ear tip and end of xiphoid process Feedback: Tube length is traditionally determined by (1) measuring the distance from the tip of the nose to the earlobe and from the earlobe to the xiphoid process, and (2) adding up to 6 inches for NG placement or at least 8 to 10 inches or more for intestinal placement, although studies do not necessarily confirm that this is a reliable technique. The physician would not prescribe a specific length and the umbilicus is not a landmark for this process. Length is not determined by aspirating from the tube.

A nurse is admitting a patient to the postsurgical unit following a gastrostomy. When planning assessments, the nurse should be aware of what potential postoperative complication of a gastrostomy? A) Premature removal of the G tube B) Bowel perforation C) Constipation D) Development of peptic ulcer disease (PUD)

Ans: A) Premature removal of the G tube Feedback: A significant postoperative complication of a gastrostomy is premature removal of the G tube. Constipation is a less immediate threat and bowel perforation and PUD are not noted to be likely complications.

A nurse is preparing to discharge a patient home on parenteral nutrition. What should an effective home care teaching program address? Select all that apply. A) Preparing the patient to troubleshoot for problems B) Teaching the patient and family strict aseptic technique C) Teaching the patient and family how to set up the infusion D) Teaching the patient to flush the line with sterile water E) Teaching the patient when it is safe to leave the access site open to air

Ans: A) Preparing the patient to troubleshoot for problems B) Teaching the patient and family strict aseptic technique C) Teaching the patient and family how to set up the infusion Feedback: An effective home care teaching program prepares the patient to store solutions, set up the infusion, flush the line with heparin, change the dressings, and troubleshoot for problems. The most common complication is sepsis. Strict aseptic technique is taught for hand hygiene, handling equipment, changing the dressing, and preparing the solution. Sterile water is never used for flushes and the access site must never be left open to air.

A patient with gastroesophageal reflux disease (GERD) has a diagnosis of Barretts esophagus with minor cell changes. Which of the following principles should be integrated into the patients subsequent care? A) The patient will require an upper endoscopy every 6 months to detect malignant changes. B) Liver enzymes must be checked regularly, as H2 receptor antagonists may cause hepatic damage. C) Small amounts of blood are likely to be present in the stools and are not cause for concern. D) Antacids may be discontinued when symptoms of heartburn subside.

Ans: A) The patient will require an upper endoscopy every 6 months to detect malignant changes Feedback: In the patient with Barretts esophagus, the cells lining the lower esophagus have undergone change and are no longer squamous cells. The altered cells are considered precancerous and are a precursor to esophageal cancer. In order to facilitate early detection of malignant cells, an upper endoscopy is recommended every 6 months. H2receptor antagonists are commonly prescribed for patients with GERD; however, monitoring of liver enzymes is not routine. Stools that contain evidence of frank bleeding or that are tarry are not expected and should be reported immediately. When antacids are prescribed for patients with GERD, they should be taken as ordered whether or not the patient is symptomatic.

A nurse is caring for a patient who has a gastrointestinal tube in place. Which of the following are indications for gastrointestinal intubation? Select all that apply. A) To remove gas from the stomach B) To administer clotting factors to treat a GI bleed C) To remove toxins from the stomach D) To open sphincters that are closed E) To diagnose GI motility disorders

Ans: A) To remove gas from the stomach C) To remove toxins from the stomach E) To diagnose GI motility disorders Feedback: GI intubation may be performed to decompress the stomach and remove gas and fluid, lavage (flush with water or other fluids) the stomach and remove ingested toxins or other harmful materials, diagnose disorders of GI motility and other disorders, administer medications and feedings, compress a bleeding site, and aspirate gastric contents for analysis. GI intubation is not used for opening sphincters that are not functional or for administering clotting factors.

A patient returns to the unit after a neck dissection. The surgeon placed a Jackson Pratt drain in the wound. When assessing the wound drainage over the first 24 postoperative hours the nurse would notify the physician immediately for what? A) Presence of small blood clots in the drainage B) 60 mL of milky or cloudy drainage C) Spots of drainage on the dressings surrounding the drain D) 120 mL of serosanguinous drainage

Ans: B) 60 mL of milky or cloudy drainage Feedback: Between 80 and 120 mL of serosanguineous secretions may drain over the first 24 hours. Milky drainage is indicative of a chyle fistula, which requires prompt treatment.

A patient who had a hemiglossectomy earlier in the day is assessed postoperatively, revealing a patent airway, stable vital signs, and no bleeding or drainage from the operative site. The nurse notes the patient is alert. What is the patients priority need at this time? A) Emotional support from visitors and staff B) An effective means of communicating with the nurse C) Referral to a speech therapist D) Dietary teaching focused on consistency of food and frequency of feedings

Ans: B) An effective means of communicating with the nurse Feedback: Verbal communication may be impaired by radical surgery for oral cancer. It is therefore vital to assess the patients ability to communicate in writing before surgery. Emotional support and dietary teaching are critical aspects of the plan of care; however, the patients ability to communicate would be essential for both. Referral to a speech therapist will be required as part of the patients rehabilitation; however, it is not a priority at this particular time. Communication with the nurse is crucial for the delivery of safe and effective care.

A patient's NG tube has become clogged after the nurse instilled a medication that was insufficiently crushed. The nurse has attempted to aspirate with a large-bore syringe, with no success. What should the nurse do next? A) Withdraw the NG tube 3 to 5 cm and reattempt aspiration. B) Attach a syringe filled with warm water and attempt an in-and-out motion of instilling and aspirating. C) Withdraw the NG tube slightly and attempt to dislodge by flicking the tube with the fingers. D) Remove the NG tube promptly and obtain an order for reinsertion from the primary care provider.

Ans: B) Attach a syringe filled with warm water and attempt an in-and-out motion of instilling and aspirating Feedback: When a tube is first noted to be clogged, a 30- to 60-mL syringe should be attached to the end of the tube and any contents aspirated and discarded. Then the syringe should be filled with warm water, attached to the tube again, and a back-and-forth motion initiated to help loosen the clog. Removal is not warranted at this early stage and a flicking motion is not recommended. The tube should not be withdrawn, even a few centimeters.

A nurse is caring for a patient who has just had a rigid fixation of a mandibular fracture. When planning the discharge teaching for this patient, what would the nurse be sure to include? A) Increasing calcium intake to promote bone healing B) Avoiding chewing food for the specified number of weeks after surgery C) Techniques for managing parenteral nutrition in the home setting D) Techniques for managing a gastrostomy

Ans: B) Avoiding chewing food for the specified number of weeks after surgery Feedback: The patient who has had rigid fixation should be instructed not to chew food in the first 1 to 4 weeks after surgery. A liquid diet is recommended, and dietary counseling should be obtained to ensure optimal caloric and protein intake. Increased calcium intake will not have an appreciable effect on healing. Enteral and parenteral nutrition are rarely necessary.

A nurse is aware of the high incidence of catheter-related bloodstream infections in patients receiving parenteral nutrition. What nursing action has the greatest potential to reduce catheter-related bloodstream infections? A) Use clean technique and wear a mask during dressing changes. B) Change the dressing no more than weekly. C) Apply antibiotic ointment around the site with each dressing change. D) Irrigate the insertion site with sterile water during each dressing change.

Ans: B) Change the dressing no more than weekly Feedback: The CDC (2011) recommends changing CVAD dressings not more than every 7 days unless the dressing is damp, bloody, loose, or soiled. Sterile technique (not clean technique) is used. Irrigation and antibiotic ointments are not used.

A nurse is creating a care plan for a patient who is receiving parenteral nutrition. The patient's care plan should include nursing actions relevant to what potential complications? Select all that apply. A) Dumping syndrome B) Clotted or displaced catheter C) Pneumothorax D) Hyperglycemia E) Line sepsis

Ans: B) Clotted or displaced catheter C) Pneumothorax D) Hyperglycemia E) Line sepsis Feedback: Common complications of PN include a clotted or displaced catheter, pneumothorax, hyperglycemia, and infection from the venous access device (line sepsis). Dumping syndrome applies to enteral nutrition, not PN.

A patient's enteral feedings have been determined to be too concentrated based on the patient's development of dumping syndrome. What physiologic phenomenon caused this patient's complication of enteral feeding? A) Increased gastric secretion of HCl and gastrin because of high osmolality of feeds B) Entry of large amounts of water into the small intestine because of osmotic pressure C) Mucosal irritation of the stomach and small intestine by the high concentration of the feed D) Acid-base imbalance resulting from the high volume of solutes in the feed

Ans: B) Entry of large amounts of water into the small intestine because of osmotic pressure Feedback: When a concentrated solution of high osmolality entering the intestines is taken in quickly or in large amounts, water moves rapidly into the intestinal lumen from fluid surrounding the organs and the vascular compartment. This results in dumping syndrome. Dumping syndrome is not the result of changes in HCl or gastrin levels. It is not caused by an acid-base imbalance or direct irritation of the GI mucosa.

A nurse is initiating parenteral nutrition (PN) to a postoperative patient who has developed complications. The nurse should initiate therapy by performing which of the following actions? A) Starting with a rapid infusion rate to meet the patient's nutritional needs as quickly as possible B) Initiating the infusion slowly and monitoring the patient's fluid and glucose tolerance C) Changing the rate of administration every 2 hours based on serum electrolyte values D) Increasing the rate of infusion at mealtimes to mimic the circadian rhythm of the body

Ans: B) Initiating the infusion slowly and monitoring the patient's fluid and glucose tolerance Feedback: PN solutions are initiated slowly and advanced gradually each day to the desired rate as the patient's fluid and glucose tolerance permits. The formulation of the PN solutions is calculated carefully each day to meet the complete nutritional needs of the individual patient based on clinical findings and laboratory data. It is not infused more quickly at mealtimes.

A patient is concerned about leakage of gastric contents out of the gastric sump tube the nurse has just inserted. What would the nurse do to prevent reflux gastric contents from coming through the blue vent of a gastric sump tube? A) Prime the tubing with 20 mL of normal saline. B) Keep the vent lumen above the patient's waist. C) Maintain the patient in a high Fowler's position. D) Have the patient pin the tube to the thigh.

Ans: B) Keep the vent lumen above the patient's waist Feedback: The blue vent lumen should be kept above the patient's waist to prevent reflux of gastric contents through it; otherwise it acts as a siphon. A one-way anti-reflux valve seated in the blue pigtail can prevent the reflux of gastric contents out the vent lumen. To prevent reflux, the nurse does not prime the tubing, maintain the patient in a high Fowler's position, or have the patient pin the tube to the thigh.

Results of a patient barium swallow suggest that the patient has GERD. The nurse is planning health education to address the patients knowledge of this new diagnosis. Which of the following should the nurse encourage? A) Eating several small meals daily rather than 3 larger meals B) Keeping the head of the bed slightly elevated C) Drinking carbonated mineral water rather than soft drinks D) Avoiding food or fluid intake after 6:00 p.m.

Ans: B) Keeping the head of the bed slightly elevated Feedback: The patient with GERD is encouraged to elevate the head of the bed on 6- to 8-inch (15- to 20-cm) blocks. Frequent meals are not specifically encouraged and the patient should avoid food and fluid within 2 hours of bedtime. All carbonated beverages should be avoided.

A patient's physician has determined that for the next 3 to 4 weeks the patient will require parenteral nutrition (PN). The nurse should anticipate the placement of what type of venous access device? A) Peripheral catheter B) Nontunneled central catheter C) Implantable port D) Tunneled central catheter

Ans: B) Nontunneled central catheter Feedback: Nontunneled central catheters are used for short-term (less than 6 weeks) IV therapy. A peripheral catheter can be used for the administration of peripheral parenteral nutrition for 5 to 7 days. Implantable ports and tunneled central catheters are for long-term use and may remain in place for many years. Peripherally inserted central catheters (PICCs) are another potential option.

The nurses comprehensive assessment of a patient includes inspection for signs of oral cancer. What assessment finding is most characteristic of oral cancer in its early stages? A) Dull pain radiating to the ears and teeth B) Presence of a painless sore with raised edges C) Areas of tenderness that make chewing difficult D) Diffuse inflammation of the buccal mucosa

Ans: B) Presence of a painless sore with raised edges Feedback: Malignant lesions of the oral cavity are most often painless lumps or sores with raised borders. Because they do not bother the patient, delay in seeking treatment occurs frequently, and negatively affects prognosis. Dull pain radiating to the ears and teeth is characteristic of malocclusion. Inflammation of the buccal mucosa causes discomfort and often occurs as a side effect of chemotherapy. Tenderness resulting in pain on chewing may be associated with gingivitis, abscess, irritation from dentures, and other causes. Pain related to oral cancer is a late symptom.

A nurse is caring for a patient who has undergone neck resection with a radial forearm free flap. The nurses most recent assessment of the graft reveals that it has a bluish color and that mottling is visible. What is the nurses most appropriate action? A) Document the findings as being consistent with a viable graft. B) Promptly report these indications of venous congestion. C) Closely monitor the patient and reassess in 30 minutes. D) Reposition the patient to promote peripheral circulation.

Ans: B) Promptly report these indications of venous congestion Feedback: A graft that is blue with mottling may indicate venous congestion. This finding constitutes a risk for tissue ischemia and necrosis; prompt referral is necessary.

A patient has been diagnosed with an esophageal diverticulum after undergoing diagnostic imaging. When taking the health history, the nurse should expect the patient to describe what sign or symptom? A) Burning pain on swallowing B) Regurgitation of undigested food C) Symptoms mimicking a heart attack D) Chronic parotid abscesses

Ans: B) Regurgitation of undigested food Feedback: An esophageal diverticulum is an outpouching of mucosa and submucosa that protrudes through the esophageal musculature. Food becomes trapped in the pouch and is frequently regurgitated when the patient assumes a recumbent position. The patient may experience difficulty swallowing; however, burning pain is not a typical finding. Symptoms mimicking a heart attack are characteristic of GERD. Chronic parotid abscesses are not associated with a diagnosis of esophageal diverticulum.

A nurse is writing a care plan for a patient with a nasogastric tube in place for gastric decompression. What risk nursing diagnosis is the most appropriate component of the care plan? A) Risk for Excess Fluid Volume Related to Enteral Feedings B) Risk for Impaired Skin Integrity Related to the Presence of NG Tube C) Risk for Unstable Blood Glucose Related to Enteral Feedings D) Risk for Impaired Verbal Communication Related to Presence of NG Tube

Ans: B) Risk for Impaired Skin Integrity Related to the Presence of NG Tube Feedback: NG tubes can easily damage the delicate mucosa of the nose, sinuses, and upper airway. An NG tube does not preclude verbal communication. This patient's NG tube is in place for decompression, so complications of enteral feeding do not apply.

A nurse is caring for a patient with a subclavian central line who is receiving parenteral nutrition (PN). In preparing a care plan for this patient, what nursing diagnosis should the nurse prioritize? A) Risk for Activity Intolerance Related to the Presence of a Subclavian Catheter B) Risk for Infection Related to the Presence of a Subclavian Catheter C) Risk for Functional Urinary Incontinence Related to the Presence of a Subclavian Catheter D) Risk for Sleep Deprivation Related to the presence of a Subclavian Catheter

Ans: B) Risk for infection related to the presence of a subclavian catheter Feedback: The high glucose content of PN solutions makes the solutions an idea culture media for bacterial and fungal growth, and the central venous access devices provide a port of entry. Prevention of infection is consequently a high priority. The patient will experience some inconveniences with regard to toileting, activity, and sleep, but the infection risk is a priority over each of these.

The nurse is preparing to insert a patient's ordered NG tube. What factor should the nurse recognize as a risk for incorrect placement? A) The patient is obese and has a "short neck." B) The patient is agitated. C) The patient has a history of gastroesophageal reflux disease (GERD). D) The patient is being treated for pneumonia.

Ans: B) The patient is agitated Feedback: Inappropriate placement may occur in patients with decreased levels of consciousness, confused mental states, poor or absent cough and gag reflexes, or agitation during insertion. A "short neck," GERD, and pneumonia are not linked to incorrect placement.

A nurse in an oral surgery practice is working with a patient scheduled for removal of an abscessed tooth. When providing discharge education, the nurse should recommend which of the following actions? A) Rinse the mouth with alcohol before bedtime for the next 7 days. B) Use warm saline to rinse the mouth as needed. C) Brush around the area with a firm toothbrush to prevent infection. D) Use a toothpick to dislodge any debris that gets lodged in the socket.

Ans: B) Use warm saline to rinse the mouth as needed Feedback: The patient should be assessed for bleeding after the tooth is extracted. The mouth can be rinsed with warm saline to keep the area clean. A firm toothbrush or toothpick could injure the tissues around the extracted area. Alcohol would injure tissues that are healing.

A patient is postoperative day 1 following gastrostomy. The nurse is planning interventions to address the nursing diagnosis of Risk for Infection Related to Presence of Wound and Tube. What intervention is most appropriate? A) Administer antibiotics via the tube as ordered. B) Wash the area around the tube with soap and water daily. C) Cleanse the skin within 2 cm of the insertion site with hydrogen peroxide once per shift. D) Irrigate the skin surrounding the insertion site with normal saline before each use.

Ans: B) Wash the area around the tube with soap and water daily Feedback: Infection can be prevented by keeping the skin near the insertion site clean using soap and water. Hydrogen peroxide is not used, due to associated skin irritation. The skin around the site is not irrigated with normal saline and antibiotics are not administered to prevent site infection.

The management of the patient's gastrostomy is an assessment priority for the home care nurse. What statement would indicate that the patient is managing the tube correctly? A) "I clean my stoma twice a day with alcohol." B) "The only time I flush my tube is when I'm putting in medications." C) "I flush my tube with water before and after each of my medications." D) "I try to stay still most of the time to avoid dislodging my tube."

Ans: C) "I flush my tube with water before and after each of my medications" Feedback: Frequent flushing is needed to prevent occlusion, and should not just be limited to times of medication administration. Alcohol will irritate skin surrounding the insertion site and activity should be maintained as much as possible.

A community health nurse serves a diverse population. What individual would likely face the highest risk for parotitis? A) A patient who is receiving intravenous antibiotic therapy in the home setting B) A patient who has a chronic venous ulcer C) An older adult whose medication regimen includes an anticholinergic D) A patient with poorly controlled diabetes who receives weekly wound care

Ans: C) An older adult whose medication regimen includes an anticholinergic Feedback: Elderly, acutely ill, or debilitated people with decreased salivary flow from general dehydration or medications are at high risk for parotitis. Anticholinergic medications inhibit saliva production. Antibiotics, diabetes, and wounds are not risk factors for parotitis.

A nurse is assessing a patient who has just been admitted to the postsurgical unit following surgical resection for the treatment of oropharyngeal cancer. What assessment should the nurse prioritize? A) Assess ability to clear oral secretions. B) Assess for signs of infection. C) Assess for a patent airway. D) Assess for ability to communicate.

Ans: C) Assess for a patent airway Feedback: Postoperatively, the nurse assesses for a patent airway. The patients ability to manage secretions has a direct bearing on airway patency. However, airway patency is the overarching goal. This immediate physiologic need is prioritized over communication, though this is an important consideration. Infection is not normally a threat in the immediate postoperative period.

A nurse is providing health promotion education to a patient diagnosed with an esophageal reflux disorder. What practice should the nurse encourage the patient to implement? A) Keep the head of the bed lowered. B) Drink a cup of hot tea before bedtime. C) Avoid carbonated drinks. D) Eat a low-protein diet.

Ans: C) Avoid carbonated drinks Feedback: For a patient diagnosed with esophageal reflux disorder, the nurse should instruct the patient to keep the head of the bed elevated. Carbonated drinks, caffeine, and tobacco should be avoided. Protein limitation is not necessary.

Prior to a patient's scheduled jejunostomy, the nurse is performing the preoperative assessment. What goal should the nurse prioritize during the preoperative assessment? A) Determining the patient's nutritional needs B) Determining that the patient fully understands the postoperative care required C) Determining the patient's ability to understand and cooperate with the procedure D) Determining the patient's ability to cope with an altered body image

Ans: C) Determining the patient's ability to understand and cooperate with the procedure Feedback: The major focus of the preoperative assessment is to determine the patient's ability both to understand and cooperate with the procedure. Body image, nutritional needs, and postoperative care are all important variables, but they are not the main focuses of assessment during the immediate preoperative period.

A patient receiving tube feedings is experiencing diarrhea. The nurse and the physician suspect that the patient is experiencing dumping syndrome. What intervention is most appropriate? A) Stop the tube feed and aspirate stomach contents. B) Increase the hourly feed rate so it finishes earlier. C) Dilute the concentration of the feeding solution. D) Administer fluid replacement by IV.

Ans: C) Dilute the concentration of the feeding solution Feedback: Dumping syndrome can generally be alleviated by starting with a dilute solution and then increasing the concentration of the solution over several days. Fluid replacement may be necessary but does not prevent or treat dumping syndrome. There is no need to aspirate stomach contents. Increasing the rate will exacerbate the problem.

A nurse is addressing the prevention of esophageal cancer in response to a question posed by a participant in a health promotion workshop. What action has the greatest potential to prevent esophageal cancer? A) Promotion of a nutrient-dense, low-fat diet B) Annual screening endoscopy for patients over 50 with a family history of esophageal cancer C) Early diagnosis and treatment of gastroesophageal reflux disease D) Adequate fluid intake and avoidance of spicy foods

Ans: C) Early diagnosis and treatment of gastroesophageal reflux disease Feedback: There are numerous risk factors for esophageal cancer but chronic esophageal irritation or GERD is among the most significant. This is a more significant risk factor than dietary habits. Screening endoscopies are not recommended solely on the basis of family history.

A nurse is providing care for a patient with a diagnosis of late-stage Alzheimer's disease. The patient has just returned to the medical unit to begin supplemental feedings through an NG tube. Which of the nurse's assessments addresses this patient's most significant potential complication of feeding? A) Frequent assessment of the patient's abdominal girth B) Assessment for hemorrhage from the nasal insertion site C) Frequent lung auscultation D) Vigilant monitoring of the frequency and character of bowel movements

Ans: C) Frequent lung auscultation Feedback: Aspiration is a risk associated with tube feeding; this risk may be exacerbated by the patient's cognitive deficits. Consequently, the nurse should auscultate the patient's lungs and monitor oxygen saturation closely. Bowel function is important, but the risk for aspiration is a priority. Hemorrhage is highly unlikely and the patient's abdominal girth is not a main focus of assessment.

The nurse is caring for a patient who is postoperative from having a gastrostomy tube placed. What should the nurse do on a daily basis to prevent skin breakdown? A) Verify tube placement. B) Loop adhesive tape around the tube and connect it securely to the abdomen. C) Gently rotate the tube. D) Change the wet-to-dry dressing.

Ans: C) Gently rotate the tube Feedback: The nurse verifies the tube's placement and gently rotates the tube once daily to prevent skin breakdown. Verifying tube placement and taping the tube to the abdomen do not prevent skin breakdown. A gastrostomy wound does not have a wet-to-dry dressing.

A critical care nurse is caring for a patient diagnosed with acute pancreatitis. The nurse knows that the indications for starting parenteral nutrition (PN) for this patient are what? A) 5% deficit in body weight compared to preillness weight and increased caloric need B) Calorie deficit and muscle wasting combined with low electrolyte levels C) Inability to take in adequate oral food or fluids within 7 days D) Significant risk of aspiration coupled with decreased level of consciousness

Ans: C) Inability to take in adequate oral food or fluids within 7 days Feedback: The indications for PN include an inability to ingest adequate oral food or fluids within 7 days. Weight loss, muscle wasting combined with electrolyte imbalances, and aspiration indicate a need for nutritional support, but this does not necessary have to be parenteral.

A patient has undergone surgery for oral cancer and has just been extubated in postanesthetic recovery. What nursing action best promotes comfort and facilitates spontaneous breathing for this patient? A) Placing the patient in a left lateral position B) Administering opioids as ordered C) Placing the patient in Fowlers position D) Teaching the patient to use the patient-controlled analgesia (PCA) system

Ans: C) Placing the patient in Fowlers position Feedback: After the endotracheal tube or airway has been removed and the effects of the anesthesia have worn off, the patient may be placed in Fowlers position to facilitate breathing and promote comfort. Lateral positioning does not facilitate oxygenation or comfort. Medications do not facilitate spontaneous breathing.

A patient who underwent surgery for esophageal cancer is admitted to the critical care unit following postanesthetic recovery. Which of the following should be included in the patients immediate postoperative plan of care? A) Teaching the patient to self-suction B) Performing chest physiotherapy to promote oxygenation C) Positioning the patient to prevent gastric reflux D) Providing a regular diet as tolerated

Ans: C) Positioning the patient to prevent gastric reflux Feedback: After recovering from the effects of anesthesia, the patient is placed in a low Fowlers position, and later in a Fowlers position, to help prevent reflux of gastric secretions. The patient is observed carefully for regurgitation and dyspnea because a common postoperative complication is aspiration pneumonia. In this period of recovery, self-suctioning is also not likely realistic or safe. Chest physiotherapy is contraindicated because of the risk of aspiration. Nutrition is prioritized, but a regular diet is contraindicated in the immediate recovery from esophageal surgery.

A patient who has had a radical neck dissection is being prepared for discharge. The discharge plan includes referral to an outpatient rehabilitation center for physical therapy. What would the goals of physical therapy for this patient include? A) Muscle training to relieve dysphagia B) Relieving nerve paralysis in the cervical plexus C) Promoting maximum shoulder function D) Alleviating achalasia by decreasing esophageal peristalsis

Ans: C) Promoting maximum shoulder function Feedback: Shoulder drop occurs as a result of radical neck dissection. Shoulder function can be improved by rehabilitation exercises. Rehabilitation would not be initiated until the patients neck incision and graft, if present, were sufficiently healed. Nerve paralysis in the cervical plexus and other variables affecting swallowing would be managed by a speech therapist rather than a physical therapist.

A patient has undergone rigid fixation for the correction of a mandibular fracture suffered in a fight. What area of care should the nurse prioritize when planning this patients discharge education? A) Resumption of activities of daily living B) Pain control C) Promotion of adequate nutrition D) Strategies for promoting communication

Ans: C) Promotion of adequate nutrition Feedback: The patient who has had rigid fixation should be instructed not to chew food in the first 1 to 4 weeks after surgery. A liquid diet is recommended, and dietary counseling should be obtained to ensure optimal caloric and protein intake. The nature of this surgery threatens the patients nutritional status; this physiologic need would likely supersede the resumption of ADLs. Pain should be under control prior to discharge and communication is not precluded by this surgery.

A patient who suffered a stroke had an NG tube inserted to facilitate feeding shortly after admission. The patient has since become comatose and the patient's family asks the nurse why the physician is recommending the removal of the patient's NG tube and the insertion of a gastrostomy tube. What is the nurse's best response? A) It eliminates the risk for infection. B) Feeds can be infused at a faster rate. C) Regurgitation and aspiration are less likely. D) It allows caregivers to provide personal hygiene more easily.

Ans: C) Regurgitation and aspiration are less likely Feedback: Gastrostomy is preferred over NG feedings in the patient who is comatose because the gastroesophageal sphincter remains intact, making regurgitation and aspiration less likely than with NG feedings. Both tubes carry a risk for infection; this change in care is not motivated by the possibility of faster infusion or easier personal care.

A patient with cancer of the tongue has had a radical neck dissection. What nursing assessment would be a priority for this patient? A) Presence of acute pain and anxiety B) Tissue integrity and color of the operative site C) Respiratory status and airway clearance D) Self-esteem and body image

Ans: C) Respiratory status and airway clearance Feedback: Postoperatively, the patient is assessed for complications such as altered respiratory status, wound infection, and hemorrhage. The other assessments are part of the plan of care for a patient who has had a radical neck dissection, but are not the nurses chief priority.

A nurse is participating in a patient's care conference and the team is deciding between parenteral nutrition (PN) and a total nutritional admixture (TNA). What advantages are associated with providing TNA rather than PN? A) TNA can be mixed by a certified registered nurse. B) TNA can be administered over 8 hours, while PN requires 24-hour administration. C) TNA is less costly than PN. D) TNA does not require the use of a micron filter.

Ans: C) TNA is less costly than PN Feedback: TNA is mixed in one container and administered to the patient over a 24-hour period. A 1.5-micron filter is used with the TNA solution. Advantages of the TNA over PN include cost savings. Pharmacy staff must prepare both solutions.

A patient with dysphagia is scheduled for PEG tube insertion and asks the nurse how the tube will stay in place. What is the nurse's best response? A) Adhesive holds a flange in place against the abdominal skin. B) A stitch holds the tube in place externally. C) The tube is stitched to the abdominal skin externally and the stomach wall internally. D) An internal retention disc secures the tube against the stomach wall.

Ans: D) An internal retention disc secures the tube against the stomach wall Feedback: A PEG tube is held in place by an internal retention disc (flange) that holds it against the stomach wall. It is not held in place by stitches or adhesives.

A radial graft is planned in the treatment of a patients oropharyngeal cancer. In order to ensure that the surgery will be successful, the care team must perform what assessment prior to surgery? A) Assessing function of cranial nerves V, VI, and IX B) Assessing for a history of GERD C) Assessing for signs or symptoms of atherosclerosis D) Assessing the patency of the ulnar artery

Ans: D) Assessing the patency of the ulnar artery Feedback: If a radial graft is to be performed, an Allen test on the donor arm must be performed to ensure that the ulnar artery is patent and can provide blood flow to the hand after removal of the radial artery. The success of this surgery is not primarily dependent on CN function or the absence of GERD and atherosclerosis.

A nurse is providing oral care to a patient who is comatose. What action best addresses the patients risk of tooth decay and plaque accumulation? A) Irrigating the mouth using a syringe filled with a bacteriocidal mouthwash B) Applying a water-soluble gel to the teeth and gums C) Wiping the teeth and gums clean with a gauze pad D) Brushing the patients teeth with a toothbrush and small amount of toothpaste

Ans: D) Brushing the patients teeth with a toothbrush and a small amount of toothpaste Feedback: Application of mechanical friction is the most effective way to cleanse the patients mouth. If the patient is unable to brush teeth, the nurse may brush them, taking precautions to prevent aspiration; or as a substitute, the nurse can achieve mechanical friction by wiping the teeth with a gauze pad. Bacteriocidal mouthwash does reduce plaque-causing bacteria; however, it is not as effective as application of mechanical friction. Water-soluble gel may be applied to lubricate dry lips, but it is not part of oral care.

A nurse is preparing to administer a patient's scheduled parenteral nutrition (PN). Upon inspecting the bag, the nurse notices that the presence of small amounts of white precipitate are present in the bag. What is the nurse's best action? A) Recognize this as an expected finding. B) Place the bag in a warm environment for 30 minutes. C) Shake the bag vigorously for 10 to 20 seconds. D) Contact the pharmacy to obtain a new bag of PN.

Ans: D) Contact the pharmacy to obtain a new bag of PN Feedback: Before PN infusion is administered, the solution must be inspected for separation, oily appearance (also known as a "cracked solution"), or any precipitate (which appears as white crystals). If any of these are present, it is not used. Warming or shaking the bag is inappropriate and unsafe.

An emergency department nurse is admitting a 3-year-old brought in after swallowing a piece from a wooden puzzle. The nurse should anticipate the administration of what medication in order to relax the esophagus to facilitate removal of the foreign body? A) Haloperidol B) Prostigmine C) Epinephrine D) Glucagon

Ans: D) Glucagon Feedback: Glucagon is administered prior to removal of a foreign body because it relaxes the smooth muscle of the esophagus, facilitating insertion of the endoscope. Haloperidol is an antipsychotic drug and is not indicated. Prostigmine is prescribed for patients with myastheniagravis. It increases muscular contraction, an effect opposite that which is desired to facilitate removal of the foreign body. Epinephrine is indicated in asthma attack and bronchospasm.

A nurse is performing health education with a patient who has a history of frequent, serious dental caries. When planning educational interventions, the nurse should identify a risk for what nursing diagnosis? A) Ineffective Tissue Perfusion B) Impaired Skin Integrity C) Aspiration D) Imbalanced Nutrition: Less Than Body Requirements

Ans: D) Imbalanced Nutrition: Less Than Body Requirements Feedback: Because digestion normally begins in the mouth, adequate nutrition is related to good dental health and the general condition of the mouth. Any discomfort or adverse condition in the oral cavity can affect a persons nutritional status. Dental caries do not typically affect the patients tissue perfusion or skin integrity. Aspiration is not a likely consequence of dental caries.

A nurse is caring for a patient who is acutely ill and has included vigilant oral care in the patients plan of care. Why are patients who are ill at increased risk for developing dental caries? A) Hormonal changes brought on by the stress response cause an acidic oral environment B) Systemic infections frequently migrate to the teeth C) Hydration that is received intravenously lacks fluoride D) Inadequate nutrition and decreased saliva production can cause cavities

Ans: D) Inadequate nutrition and decreased saliva production can cause cavities Feedback: Many ill patients do not eat adequate amounts of food and therefore produce less saliva, which in turn reduces the natural cleaning of the teeth. Stress response is not a factor, infections generally do not attack the enamel of the teeth, and the fluoride level of the patient is not significant in the development of dental caries in the ill patient.

A patient seeking care because of recurrent heartburn and regurgitation is subsequently diagnosed with a hiatal hernia. Which of the following should the nurse include in health education? A) Drinking beverages after your meal, rather than with your meal, may bring some relief. B) Its best to avoid dry foods, such as rice and chicken, because theyre harder to swallow. C) Many patients obtain relief by taking over-the-counter antacids 30 minutes before eating. D) Instead of eating three meals a day, try eating smaller amounts more often.

Ans: D) Instead of eating three meals a day, try eating smaller amounts more often Feedback: Management for a hiatal hernia includes frequent, small feedings that can pass easily through the esophagus. Avoiding beverages and particular foods or taking OTC antacids are not noted to be beneficial.

A patient has been diagnosed with a malignancy of the oral cavity and is undergoing oncologic treatment. The oncologic nurse is aware that the prognosis for recovery from head and neck cancers is often poor because of what characteristic of these malignancies? A) Radiation therapy often results in secondary brain tumors. B) Surgical complications are exceedingly common. C) Diagnosis rarely occurs until the cancer is endstage. D) Metastases are common and respond poorly to treatment.

Ans: D) Metastases are common and respond poorly to treatment Feedback: Deaths from malignancies of the head and neck are primarily attributable to local-regional metastasis to the cervical lymph nodes in the neck. This often occurs by way of the lymphatics before the primary lesion has been treated. This local-regional metastasis is not amenable to surgical resection and responds poorly to chemotherapy and radiation therapy. This high mortality rate is not related to surgical complications, late diagnosis, or the development of brain tumors.

A patient has been brought to the emergency department by EMS after telling a family member that he deliberately took an overdose of NSAIDs a few minutes earlier. If lavage is ordered, the nurse should prepare to assist with the insertion of what type of tube? A) Nasogastric tube B) Levin tube C) Gastric sump D) Orogastric tube

Ans: D) Orogastric tube Feedback: An orogastric tube is a large-bore tube inserted through the mouth with a wide outlet for removal of gastric contents; it is used primarily in the emergency department or an intensive care setting. Nasogastric, Levin, and gastric sump tubes are not for this specific purpose.

A nurse is caring for a patient with a nasogastric tube for feeding. During shift assessment, the nurse auscultates a new onset of bilateral lung crackles and notes a respiratory rate of 30 breaths per minute. The patient's oxygen saturation is 89% by pulse oximetry. After ensuring the patient's immediate safety, what is the nurse's most appropriate action? A) Perform chest physiotherapy. B) Reduce the height of the patient's bed and remove the NG tube. C) Liaise with the dietitian to obtain a feeding solution with lower osmolarity. D) Report possible signs of aspiration pneumonia to the primary care provider.

Ans: D) Report possible signs of aspiration pneumonia to the primary care provider Feedback: The patient should be assessed for further signs of aspiration pneumonia. It is unnecessary to remove the NG tube and chest physiotherapy is not indicated. A different feeding solution will not resolve this complication.

After 20 seconds of auscultating for bowel sounds on a client recovering from abdominal surgery, the nurse hears nothing. Which of the following should the nurse do based on the assessment findings?

Auscultation is used to determine the character, location, and frequency of bowel sounds. The frequency and character of sounds are usually heard as clicks and gurgles that occur irregularly and range from 5 to 35 per minutes. Normal sounds are heard about every 5 to 20 seconds, whereas hypoactive sounds can be one or two sounds in 2 minutes. Postoperatively, it is common for sounds to be reduced; therefore, the nurse needs to listen at least 3 to 5 minutes to verify absent or no bowel sounds.

24. A patient who is undergoing consolidation therapy for the treatment of leukemia has been experiencing debilitating fatigue. How can the nurse best meet this patient's needs for physical activity? A) Teach the patient about the risks of immobility and the benefits of exercise. B) Assist the patient to a chair during awake times, as tolerated. C) Collaborate with the physical therapist to arrange for stair exercises. D) Teach the patient to perform deep breathing and coughing exercises.

B) Assist the patient to a chair during awake times, as tolerated. Sitting is a chair is preferable to bed rest, even if a patient is experiencing severe fatigue. A patient who has debilitating fatigue would not likely be able to perform stair exercises. Teaching about mobility may be necessary, but education must be followed by interventions that actually involve mobility. Deep breathing and coughing reduce the risk of respiratory complications but are not substitutes for physical mobility in preventing deconditioning.

19. A nurse is caring for a patient who has been diagnosed with leukemia. The nurse's most recent assessment reveals the presence of ecchymoseson the patient's sacral area and petechiae in her forearms. In addition to informing the patient's primary care provider, the nurse should perform what action? A) Initiate measures to prevent venous thromboembolism (VTE). B) Check the patient's most recent platelet level. C) Place the patient on protective isolation. D) Ambulate the patient to promote circulatory function.

B) Check the patient's most recent platelet level. The patient's signs are suggestive of thrombocytopenia, thus the nurse should check the patient's most recent platelet level. VTE is not a risk and this does not constitute a need for isolation. Ambulation and activity may be contraindicated due to the risk of bleeding.

16. A nurse practitioner is assessing a patient who has a fever, malaise, and a white blood cell count that is elevated. Which of the following principles should guide the nurse's management of the patient's care? A) There is a need for the patient to be assessed for lymphoma. B) Infection is the most likely cause of the patient's change in health status. C) The patient is exhibiting signs and symptoms of leukemia. D) The patient should undergo diagnostic testing for multiple myeloma

B) Infection is the most likely cause of the patient's change in health status. Leukocytosis is most often the result of infection. It is only considered pathologic (and suggestive of leukemia) if it is persistent and extreme. Multiple myeloma and lymphoma are not likely causes of this constellation of symptoms.

15. An adult patient has presented to the health clinic with a complaint of a firm, painless cervical lymph node. The patient denies any recent infectious diseases. What is the nurse's most appropriate response to the patient's complaint? A) Call 911. B) Promptly refer the patient for medical assessment. C) Facilitate a radiograph of the patient's neck and have the results forwarded to the patient's primary care provider. D) Encourage the patient to track the size of the lymph node and seek care in 1 week.

B) Promptly refer the patient for medical assessment Hodgkin lymphoma usually begins as an enlargement of one or more lymph nodes on one side of the neck. The individual nodes are painless and firm but not hard. Prompt medical assessment is necessary if a patient has this presentation. However, there is no acute need to call 911. Delaying care for 1 week could have serious consequences and x-rays are not among the common diagnostic tests.

36. An adult patient's abnormal complete blood count (CBC) and physical assessment have prompted the primary care provider to order a diagnostic workup for Hodgkin lymphoma. The presence of what assessment finding is considered diagnostic of the disease? A) Schwann cells B) Reed-Sternberg cells C) Lewy bodies D) Loops of Henle

B) Reed-Sternberg cells The malignant cell of Hodgkin lymphoma is the Reed-Sternberg cell, a gigantic tumor cell that is morphologically unique and thought to be of immature lymphoid origin. It is the pathologic hallmark and essential diagnostic criterion. Schwann cells exist in the peripheral nervous system and Lewy bodies are markers of Parkinson disease. Loops of Henle exist in nephrons.

6. A nursing student is caring for a patient with acute myeloid leukemia who is preparing to undergo induction therapy. In preparing a plan of care for this patient, the student should assign the highest priority to which nursing diagnoses? A) Activity Intolerance B) Risk for Infection C) Acute Confusion D) Risk for Spiritual Distress

B) Risk for Infection Induction therapy places the patient at risk for infection, thus this is the priority nursing diagnosis. During the time of induction therapy, the patient is very ill, with bacterial, fungal, and occasional viral infections; bleeding and severe mucositis, which causes diarrhea; and marked decline in the ability to maintain adequate nutrition. Supportive care consists of administering blood products and promptly treating infections. Immobility, confusion, and spiritual distress are possible, but infection is the patient's most acute physiologic threat.

5. A patient with advanced leukemia is responding poorly to treatment. The nurse finds the patient tearful and trying to express his feelings, but he is clearly having difficulty. What is the nurse's most appropriate action? A) Tell him that you will give him privacy and leave the room. B) Offer to call pastoral care. C) Ask if he would like you to sit with him while he collects his thoughts. D) Tell him that you can understand how he's feeling.

C) Ask if he would like you to sit with him while he collects his thoughts. Providing emotional support and discussing the uncertain future are crucial. Leaving is incorrect because leaving the patient doesn't show acceptance of his feelings. Offering to call pastoral care may be helpful for some patients but should be done after the nurse has spent time with the patient. Telling the patient that you understand how he's feeling is inappropriate because it doesn't help him express his feelings.

26. After receiving a diagnosis of acute lymphocytic leukemia, a patient is visibly distraught, stating, I have no idea where to go from here. How should the nurse prepare to meet this patient's psychosocial needs? A) Assess the patient's previous experience with the health care system. B) Reassure the patient that treatment will be challenging but successful. C) Assess the patient's specific needs for education and support. D) Identify the patient's plan of medical care.

C) Assess the patient's specific needs for education and support. In order to meets the patient's needs, the nurse must first identify the specific nature of these needs. According to the nursing process, assessment must precede interventions. The plan of medical care is important, but not central to the provision of support. The patient's previous health care is not a primary consideration, and the nurse cannot assure the patient of successful treatment

4. A nurse is planning the care of a patient who has been admitted to the medical unit with a diagnosis of multiple myeloma. In the patient's care plan, the nurse has identified a diagnosis of Risk for Injury. What pathophysiologic effect of multiple myeloma most contributes to this risk? A) Labyrinthitis B) Left ventricular hypertrophy C) Decreased bone density D) Hypercoagulation

C) Decreased bone density Clients with multiple myeloma are at risk for pathologic bone fractures secondary to diffuse osteoporosis and osteolytic lesions. Labyrinthitis is uncharacteristic, and patients do not normally experience hypercoagulation or cardiac hypertrophy.

7. A 77-year-old male is admitted to a unit with a suspected diagnosis of acute myeloid leukemia (AML). When planning this patient's care, the nurse should be aware of what epidemiologic fact? A) Early diagnosis is associated with good outcomes. B) Five-year survival for older adults is approximately 50%. C) Five-year survival for patients over 75 years old is less than 2%. D) Survival rates are wholly dependent on the patient's pre-illness level of health.

C) Five-year survival for patients over 75 years old is less than 2%. he 5-year survival rate for patients with AML who are 50 years of age or younger is 43%; it drops to 19% for those between 50 and 64 years, and drops to1.6% for those older than 75 years. Early diagnosis is beneficial, but is nonetheless not associated with good outcomes or high survival rates. Preillness health is significant, but not the most important variable.

23. A nurse is caring for a patient who is being treated for leukemia in the hospital. The patient was able to maintain her nutritional status for the first few weeks following her diagnosis but is now exhibiting early signs and symptoms of malnutrition. In collaboration with the dietitian, the nurse should implement what intervention? A) Arrange for total parenteral nutrition (TPN). B) Facilitate placement of a percutaneous endoscopic gastrostomy (PEG) tube. C) Provide the patient with several small, soft-textured meals each day. D) Assign responsibility for the patient's nutrition to the patient's friends and family.

C) Provide the patient with several small, soft-textured meals each day. For patients experiencing difficulties with oral intake, the provision of small, easily chewed meals may be beneficial. This option would be trialed before resorting to tube feeding or TPN. The family should be encouraged to participate in care, but should not be assigned full responsibility.

32. A clinic nurse is working with a patient who has a long-standing diagnosis of polycythemia vera. How can the nurse best gauge the course of the patient's disease? A) Document the color of the patient's palms and face during each visit. B) Follow the patient's erythrocyte sedimentation rate over time. C) Document the patient's response to erythropoietin injections. D) Follow the trends of the patient's hematocrit.

D) Follow the trends of the patient's hematocrit . The course of polycythemia vera can be best ascertained by monitoring the patient's hematocrit, which should remain below 45%. Erythropoietin injections would exacerbate the condition. Skin tone should be observed, but is a subjective assessment finding. The patient's ESR is not relevant to the course of the disease.

The client describes a test previously completed to detect a small bowel obstruction prior to admission to the hospital. The client states that the test involved insertion of a tube through the nose and lasted over 6 hours. The nurse determines which test name should be documented?

Enteroclysis is a double contrast study where a duodenal tube is inserted and 500 to 1000 mL of thin barium sulfate suspension and then methylcellulose is infused. Fluoroscopy is used to visualize the filling of the intestinal loops over a period of up to 6 hours. The test is used for detection of small bowel obstruction and diverticuli. Abdominal ultrasound, magnetic resonance imaging, and positron emission tomography do not involve insertion of a duodenal tube.

The nurse is preparing to examine the abdomen of a client complaining of a change in his bowel pattern. The nurse would place the client in which position?

When examining the abdomen, the client lies supine with his knees flexed. This position assists in relaxing the abdominal muscles. The lithotomy position commonly is used for a female pelvic examination and to examine the rectum. The knee-chest position can be used for a variety of examinations, most commonly the anus and rectum. The left Sim's lateral position may be used to assess the rectum or vagina and to administer an enema.


Conjuntos de estudio relacionados

Chapter 32: Drug Therapy to Decrease Histamine Effects and Allergic Response

View Set

A+ 2.9 Given a scenario, use appropriate networking tools

View Set

Human Growth and Development Final (Chapter 15-20)

View Set

#1 Chapter 37: Assessment and Management of Patients With Allergic Disorders

View Set